Đến nội dung

JUV nội dung

Có 136 mục bởi JUV (Tìm giới hạn từ 06-06-2020)



Sắp theo                Sắp xếp  

#635724 Tìm các số nguyên dương $x,y,z$ sao cho $\frac{x^...

Đã gửi bởi JUV on 26-05-2016 - 18:42 trong Số học

Phương trình $x^{3}+px+q=0$ có biệt thức  $27q^{2}+4p^{3}$ ở đây $p=-(tyz)^{2},q=y^{3}+z^{3}$

Mình cũng không rành lắm về biệt thức bậc 3 nhưng mình không nghĩ rằng việc phương trình có nghiệm hay không không phụ thuộc vào nó bởi vì phương trình bậc 3 có bậc lẻ nên nó luôn luôn có nghiệm dù hệ số như thế nào 




#635828 Marathon Tổ hợp và rời rạc VMF

Đã gửi bởi JUV on 27-05-2016 - 00:29 trong Tổ hợp và rời rạc

Lời giải bài 5:

 

Chọn ra 2 số thuộc $a,b$ thuộc $S$ mà $a,b$ phân biệt mà $2a\leq n< 2b$. Xét 1 cặp $(a;b)$ như thế, ta thấy rằng chỉ có 1 cấp số cộng cực đại duy nhất chứa 2 số $a$ và $b$ đó mà không chứa bất kì số nào nằm giữa 2 số đó. Tương tự, 2 cấp số cộng cực đại công sai khác 0 mà có chứa 2 số (a;b) thoả mãn  $2a< n\leq 2b$ và không chứa bất kì số nào nằm giữa 2 số đó thì sẽ trùng nhau. Vì vậy có thể thiết lập 1 song ánh giữa tập các cặp $(a;b)$ vào tập các csc cực đại công sai khác 0.Mà có thể tạo được $n$ csc cực đại công sai =0 nên số các csc cực đại bằng số cách chọn cặp $(a;b)$ cộng thêm $n$ và sẽ bằng : $(\frac{n}{2})(\frac{n}{2})+n$ nếu $n$ chẵn và bằng $(\frac{n-1}{2}+1)(\frac{n-1}{2})+n$ nếu $n$ lẻ.

Bài 6:

Cho $n$ hòn sỏi được đặt vào $n$ điểm trên 1 hình tròn. Mỗi lần ta dịch chuyển 2 hòn sỏi , mỗi hòn sỏi được dịch chuyển vào 1 trong 2 điểm mà nằm kề với hòn sỏi đó nhưng chiều dịch chuyển của 2 hòn sỏi đó ngược nhau (có 2 hướng dịch chuyển là cùng và ngược chiều kim đồng hồ). Ta muốn dịch chuyển tất cả các hòn sỏi vào 1 điểm. CMR:

a/ Ta có thể đạt được mục đích nếu $n$ lẻ

b/ Ta không thể đạt được mục đích nếu $n$ chẵn




#635835 Marathon Tổ hợp và rời rạc VMF

Đã gửi bởi JUV on 27-05-2016 - 00:49 trong Tổ hợp và rời rạc

Trường hợp n=3 thì có 1 csc nhưng theo kq của bạn thì có 2

Có 2 mà:(1;3),(1;2;3)




#635836 Marathon Tổ hợp và rời rạc VMF

Đã gửi bởi JUV on 27-05-2016 - 00:52 trong Tổ hợp và rời rạc

Mà tiện hỏi luôn là công sai có cần dương không? Nếu không thì mình sẽ cộng thêm $n$ vào kết quả




#635840 Marathon Tổ hợp và rời rạc VMF

Đã gửi bởi JUV on 27-05-2016 - 00:59 trong Tổ hợp và rời rạc

mình đọc cả 2 cách thì thấy cách của JUV bị thừa mất 1 số TH CSC chỉ có 2 số nhưng mà mình thấy với n đủ lớn thì đáp số của Long Phi phải lớn hơn của JUV không biết 2 bạn ai có nhầm ở đâu không(mình đọc thấy đúng cả)?

Mình đã sửa lại rồi, mình tưởng là công sai phải dương nên xét thiếu TH, mà theo định nghĩa csc thì đâu cần phải có $3$ số :https://vi.wikipedia...iki/Cấp_số_cộng




#635874 Marathon Tổ hợp và rời rạc VMF

Đã gửi bởi JUV on 27-05-2016 - 09:43 trong Tổ hợp và rời rạc

Lời giải bài 7:

 

Xét $n$ chia hết cho 3, khi đó chia bảng $2n\times 2n$ thành các bảng $3\times 3$. Từ mỗi bảng $3\times 3$ đó có thể tạo được 1 bàn cờ vua $3\times 3$ để ghép lại thành 1 bàn cờ vua $2n\times 2n$

Xét $n$ không chia hết cho 3, giả sử bảng $2n\times 2n$ ban đầu có thể biến đổi thành 1 bàn cờ vua. Nếu thế thì khi biến đổi được bảng về 1 bảng cờ vua thì luôn có 2 trong 4 góc bảng màu trắng (Vì $2n$ là số chẵn). Giả sử góc trên cùng bên trái màu trắng sau khi ta lập được 1 bàn cờ vua. Ta sẽ đánh số 1,2,3 vào các ô của bảng thoả mãn điều kiện: 

-Ô bên trái trên cùng đánh số 1

-Ô kề trái những ô đánh số 1,2,3 thì được đánh số lần lượt là 2,3,1. Nếu như ô cuối của hàng được đánh số 1,2,3 thì ô đầu tiên của hàng bên dưới hàng đó được đánh số lần lượt là 2,3,1.

Vì $n$ không chia hết cho 3 nên 3 ô liên tiếp nằm trên cùng 1 hàng hoặc cột được đánh cả 3 số 1,2,3. Vì vậy mỗi lần đổi màu thì số ô đen được đánh dấu số 1 hoặc là tăng lên 1, hoặc là giảm 1. Tương tự với các ô đánh số 2,3. Vì vậy số các ô được đánh số 1,2,3 sau lần đổi màu thứ $n+1$ lần lượt khác tính chẵn lẻ với số các ô được đánh số 1,2,3 sau lần đổi màu thứ $n$.Vì vậy tại mọi thời điểm, số các ô dược đánh số 1,2,3 luôn cùng tính chẵn lẻ với nhau. Nhưng nếu xét bàn cờ vua đã cho,vì ô trên cùng bên trái màu trắng nên số các ô được đánh số 1 là $\frac{2n^2-2}{3}$, số các ô được đánh số 2 và 3 là $\frac{2n^2-2}{3}+1$. 3 số này không cùng tính chẵn lẻ nên suy ra giả sử sai

 

Bài 8: Anne và Alice ở 2 phòng khác nhau. Alice đặt 13 quân bài từ Át đến K vào 14 ô trống nằm trên một đường tròn và có 1 ô không đặt lá bài nào, Anne không thể thấy được Alice sắp xếp thế nào. Mỗi lượt Anne sẽ gọi tên 1 lá bài. Nếu lá bài đó nằm cạnh ô trống thì Alice sẽ dịch chuyển lá bài đó về phía ô trống, còn nếu không thì không làm gì cả.Sau 1 số bước thì Anne sẽ dùng hỏi bài.

a/ Anne có một cách nào đó để chắc chắn sau khi dừng hỏi bài thì tất cả các quân bài không còn nằm ở vị trí ban đầu của nó nữa không?

b/  Anne có một cách nào đó để sau chắc chắn sau khi dừng hỏi bài thì quân Át không nằm cạnh ô trống không?




#635897 Marathon Tổ hợp và rời rạc VMF

Đã gửi bởi JUV on 27-05-2016 - 10:51 trong Tổ hợp và rời rạc

Anne không biết các quân bài được sắp xếp thế nào bạn ạ ( vì ngồi ở $2$ phòng khác nhau)




#635908 Marathon Tổ hợp và rời rạc VMF

Đã gửi bởi JUV on 27-05-2016 - 12:14 trong Tổ hợp và rời rạc

Lời giải bài 8.


 

a/ thay các lá bài bằng các số từ $1$ đến $13$ . Anne sẽ gọi lần lượt từ số $1$ tới số $13$, lặp lại một số vòng gọi như vậy. Giả sử ô trống nằm giữa $2$ số $i<j$ thì khi gọi đến $i$,số $i$ sẽ chuyển vào chỗ trống. Khi đó sẽ có 2 số cạnh ô trống là $i$ và một số $k$ nữa. Nếu $k>i$ thì khi gọi đến $k$ $k$ k sẽ di chuyển vào ô trống, và $i$ bh ko cạnh ô trống nữa, còn nếu $k<i$ thì $k$ sẽ dịch chuyển vào ô trống trong vòng gọi thứ 2. Tóm lại $i$ sẽ ko ở cạnh ô trống nữa và sẽ có số khác nhảy vào ô trống. Như vậy sau 1 số bước thì các quân bài ko còn ở vị trí ban đầu nữa.

b/Để quân Át, tức số $1$ ko năm cạnh ô trống thì Anne chỉ việc gọi như phần a/ nhưng sẽ bắt đầu gọi từ $2$ đến $13$. Số $1$ sẽ đứng im còn ô trống sẽ thay đổi nên sẽ có lúc nào đó số $1$ ko nằm cạnh ô trống.

Bạn phải chỉ ra xem Anne sẽ chắc chắn các quân bài không ở vị trí đầu sau bao nhiêu bước bởi nếu có 1 quân bài dịch chuyển hết vòng tròn thì sẽ trở lại VT đầu

Mà đáp án câu b là không thể nhé, thử kiểm tra lại suy luận của mình xem, phải  chỉ ra sau một số bước xác định để quân át không nằm cạnh ô trống chứ không phải là chỉ ra tồn tại




#636368 Marathon Tổ hợp và rời rạc VMF

Đã gửi bởi JUV on 28-05-2016 - 21:36 trong Tổ hợp và rời rạc

À sr, ý b bị xoá mất rồi, nếu như không ai làm được thì mình có thể gợi ý như sau:

-Đầu tiên xét $a_n$ là số các dãy nhị phân thoả mãn dãy đó có độ dài $n$ và số 1 đứng ở đầu trái của dãy và các khoảng cách nhị phân không quá 3. Ta có khoảng cách giữa số 1 thứ nhất và số 1 thứ 2 không quá 3 nên có 3 cách chọn số 1 thứ 2, từ đó có dãy truy hồi:

$a_{n+4}=a_{n+3}+a_{n+2}+a_{n+1}+a_n$ với $a_{1}=1;a_{2}=2,a_{3}=4,a_{4}=8$ (1)

-Tiếp theo xét các dãy nhị phân có dạng $x_{2}=101010...101$ với độ dài $n$ ( n là số lẻ). Ta đếm các số thoả mãn đề bài mà không vượt quá $x$ mà biểu diễn nhị phân của nó có độ dài $n$ , gọi số các số đó là $b_n$. Vì số 1 đầu tiên đã xác định nên có 4 cách chọn số 1 thứ 2:

-Nếu số 1 thứ 2 nằm ngay sau số 1 thứ nhất thì không có cách chọn dãy nhị phân thoả mãn

-Nếu nằm cách số 1 thứ nhất 1 số 0 thì có $b_{n-2}$ cách chọn dãy nhị phân thoả mãn vì tất cả các dãy nhị phân đó đều lớn hơn $x$

-Nếu nằm cách số 1 thứ nhất 2 số 0 thì có $a_{n-3}$ cách chọn dãy nhị phân thoả mãn

-Tương tự, nếu cách 3 số 0 thì có $a_{n-4}$ cách chọn dãy nhị phân thoả mãn

Ta có dãy truy hồi $b_{n}=b_{n-2}+a_{n-3}+a_{n-4}$

Từ đó cộng thêm kết quả câu a thì dễ dàng tìm ra đáp số (Câu a là $3095$ nhé, còn câu b là $3829$ )

P/s: Từ (1) ta cũng có thể dễ dàng suy ra câu a. Mà bài 8 của mình sao vẫn chưa có ai giải vậy  :mellow:




#636395 Marathon Tổ hợp và rời rạc VMF

Đã gửi bởi JUV on 28-05-2016 - 22:33 trong Tổ hợp và rời rạc

Được sự cho phép của bangbang1412, mình sẽ đề xuất tiếp bài $11$

Bài 11: Trên $1$ đoạn thẳng từ trái sang phải có đặt $21$ hộp gỗ, trong mỗi hộp gỗ có đặt $1$ vài hòn bi. Biết rằng số hòn bi trong mỗi hộp gỗ nhận $1$ trong các giá trị từ $1$ đến $21$ và không có $2$ hộp nào chứa cùng số bi. Mỗi bước ta sẽ chọn $2$ hộp gỗ bất kì và nếu $1$ hộp chứa nhiều bi hơn hộp kia thì ta sẽ chuyển từ hộp nhiều bi hơn sang hộp ít bi hơn $1$ số bi đúng bằng số bi mà hộp ít bi hơn có trước khi chuyển bi. Hỏi có thể luôn chuyển được bi sao cho sau các bước chuyển đó,$ 21$ hộp có số bi nhận $1$ trong các giá trị từ $1$ đến $21$, không có $2$ hộp nào có cùng số bi và số bi của các hộp từ trái sang phải sắp xếp theo thứ tự tăng dần?




#636551 Macedonia TST 2016

Đã gửi bởi JUV on 29-05-2016 - 16:02 trong Thi HSG Quốc gia và Quốc tế

Lời giải bài 2 mình đã đăng trên topic Marathon tổ hợp rời rạc rồi, mình sẽ đăng lại lên đây cho bạn nào chưa biết

Xét $n$ chia hết cho 3, khi đó chia bảng $2n\times 2n$ thành các bảng $3\times 3$. Từ mỗi bảng $3\times 3$ đó có thể tạo được 1 bàn cờ vua $3\times 3$ để ghép lại thành 1 bàn cờ vua $2n\times 2n$

Xét $n$ không chia hết cho 3, giả sử bảng $2n\times 2n$ ban đầu có thể biến đổi thành 1 bàn cờ vua. Nếu thế thì khi biến đổi được bảng về 1 bảng cờ vua thì luôn có 2 trong 4 góc bảng màu trắng (Vì $2n$ là số chẵn). Giả sử góc trên cùng bên trái màu trắng sau khi ta lập được 1 bàn cờ vua. Ta sẽ đánh số 1,2,3 vào các ô của bảng thoả mãn điều kiện: 

-Ô bên trái trên cùng đánh số 1

-Ô kề trái những ô đánh số 1,2,3 thì được đánh số lần lượt là 2,3,1. Nếu như ô cuối của hàng được đánh số 1,2,3 thì ô đầu tiên của hàng bên dưới hàng đó được đánh số lần lượt là 2,3,1.

Vì $n$ không chia hết cho 3 nên 3 ô liên tiếp nằm trên cùng 1 hàng hoặc cột được đánh cả 3 số 1,2,3. Vì vậy mỗi lần đổi màu thì số ô đen được đánh dấu số 1 hoặc là tăng lên 1, hoặc là giảm 1. Tương tự với các ô đánh số 2,3. Vì vậy số các ô được đánh số 1,2,3 sau lần đổi màu thứ $n+1$ lần lượt khác tính chẵn lẻ với số các ô được đánh số 1,2,3 sau lần đổi màu thứ $n$.Vì vậy tại mọi thời điểm, số các ô dược đánh số 1,2,3 luôn cùng tính chẵn lẻ với nhau. Nhưng nếu xét bàn cờ vua đã cho,vì ô trên cùng bên trái màu trắng nên số các ô được đánh số 1 là $\frac{2n^2-2}{3}$, số các ô được đánh số 2 và 3 là $\frac{2n^2-2}{3}+1$. 3 số này không cùng tính chẵn lẻ nên suy ra giả sử sai




#637256 $\frac{1}{f(x)}+\frac{1}{f(...

Đã gửi bởi JUV on 31-05-2016 - 22:20 trong Phương trình hàm

Đầu tiên thế $x=0$,$x=-1$ thì ta có $f(0)=f(-1)=2$. Giả sử hàm không phải hàm hằng trên đoạn $[-1;0]$, nếu tồn tại 1 số $a$ thoả mãn mãn $-1<a<0$ sao cho tất cả mọi số $b$ nằm trong khoảng $(a;0)$ đều có $f(b)$ khác 2 thì theo tính liên tục của hàm số thì với mọi $b$ nằm trong khoảng đó thì $f(b)$ hoặc lớn hơn $2$, hoặc nhỏ hơn $2$.Nếu $f(b)>2$, xét $x\in (a;0)$, khi chọn $x$ tiến đủ gần đến 0 sao cho $x^2+2x$ nằm trong khoảng $(a;0)$ thì lúc đó $f(x)>2$,$f(x^2+x)>2$ nên $\frac{1}{f(x)}+\frac{1}{f(x^{2}+2x)}<1$(vô lí).Nếu $f(b)<2$ thì lúc đó theo tính liên tục của hàm số thì tồn tại $c\in (a;0)$ sao cho $c<0$ và với mọi $b\in (c;0)$ thì $2>f(b)>0$.

Xét $x\in (c;0)$, chọn $x$ tiến đủ gần về $0$ sao cho $x^2+2x\in (c;0)$, lúc đó $0<f(x)<2$,$0<f(x^2+2x)<2$, vì vậy $\frac{1}{f(x)}+\frac{1}{f(x^{2}+2x)}>1$(vô lí). Vậy không tồn tại số $a$ thoả mãn. Vì vậy tồn tại $-1<d<0$ sao cho với mọi $b\in (d;0)$  thì $f(b)=2$. Vì $\frac{1}{f(x)}+\frac{1}{f(x^{2}+2x)}=1$ nên nếu với mọi $b\in (d;0)$ thì $f(b^2+2b)=2$ .Vậy $f(x)=2$ với mọi $x\in (d^2+2d;0)$ nếu $f(x)=2$ với mọi $x\in (d;0)$. Lặp lại quá trình đó vô hạn lần thì ta sẽ chứng minh được $f(b)=2$ với mọi $b\in (-1;0)$. Xét$x>0$, ta có thể chứng minh tương tự thì với mọi $x>0$ thì $f(x)=2$. Vậy $f(x)=2$ với $x\geq -1$. Lại có $\frac{1}{f(x)}+\frac{1}{f(x^{2}+2x)}=1$ và $x^2+2x\geq -1$ với mọi $x$ nên $\frac{1}{f(x^{2}+2x)}=2$ với mọi $x$. Vậy $f(x)=2$ là hàm cần tìm




#637324 Marathon Tổ hợp và rời rạc VMF

Đã gửi bởi JUV on 01-06-2016 - 08:53 trong Tổ hợp và rời rạc

Lời giải bài 8:

a/ Anne có thể gọi tên bài để đạt được mục đích: cô sẽ gọi $13$ lần, mỗi lần sẽ gọi lần lượt tên các lá bài từ Át đến K.Xét 1 lần gọi bất kì, không mất tính tổng quát, giả sử quân bài đầu tiên được di chuyển di chuyển theo chiều kim đồng hồ thì lúc đó quân bài thứ 2 được di chuyển là quân bài nằm cạnh ô trống và khác quân bài thứ nhất. Vì vậy quân bài đó sẽ di chuyển theo chiều kim đồng hồ, tương tự với lá bài thứ 3,4,.... Vì vậy mỗi quân bài được di chuyển sau mỗi lần đọc bài thì sẽ di chuyển theo 1 chiều nhất định. Xét sau lần đọc 1, xét 2 quân bài $a$,$b$ nằm cạnh ô trống. Nếu quân $b$ nằm bên phải, $a$ nằm bên trái ô trống thì nghĩa là trong lần đọc đầu, quân $a$ được di chuyển và quân $b$ không được di chuyển. Tất nhiên là quân $b$ được đọc trước $a$ vài nếu không thì nó sẽ nằm bên phải ô trống. Vì quân $b$ đọc trước $a$ nên trong lần đọc thứ 2 thì nó sẽ di chuyển về phía ô trống. Vì vậy sau mỗi lần đọc, có ít nhất $1$ quân không được di chuyển từ những lần trước được phép di chuyển theo chiều kim đồng hồ. Vì có $13$ lần đọc nên cả $13$ lá bài đều được di chuyển theo chiều kim đồng hồ ít nhất $1$ lần và vì trong mỗi lần, mỗi quân bài chỉ di chuyển tối đa 1 lần nên trong cả $13$ lần, $13$ quân bài di chuyển theo chiều kim đồng hồ nhiều nhất $13$ lần và ít nhất $1$ lần nên sau $13$ lần đọc thì các quân bài sẽ không còn ở vị trí cũ.

b/ Anne không thể đạt được mục đích nếu không có may mắn. Thât vậy nếu Alice sắp xếp bộ bài sao cho quân Át nằm cạnh ô trống, gọi đó là trạng thái 0 và Anne đưa Alice 1 tờ giấy ghi những lá bài mình sẽ gọi theo thứ tự nhất định từ trái sang phải cho đến khi dừng gọi. Alice sẽ ghi lại các quân bài đó trên bảng theo thứ tự ngược lại ( ví dụ như nếu Anne ghi $A,5,Q,K$ thì Alice sẽ ghi $K,Q,5,A$) và thực hiện các bước:

Gọi $n$ là số các lá bài được ghi trên giấy. Anne sẽ nhìn lên bảng và trong bước thứ $i$, xét quân bài thứ $i$ từ trái sang phải. Nếu lúc này quân bài đó nằm cạnh ô trống thì dịch chuyển quân bài đó về ô trống, còn nếu không thì không làm gì cả và chuyển qua bước $i+1$. Gọi trạng thái $i$ là cách sắp xếp bộ bài sau bước thứ $i$ thì cuối cùng sau $n$ bước thì bộ bài sẽ đạt trạng thái $n$. Ta chứng minh rằng nếu bộ bài được sắp xếp thế này và Anne đọc đúng hững quân bài được viết trên giấy theo đúng thứ tự thì cuối cùng, quân Át sẽ nằm cạnh ô trống. Ta sẽ chứng minh rằng sau lần đọc thứ $i$ thì bộ bài của $Anne$ sẽ trở về trạng thái $n-i$. Ta thấy trong lần đọc thứ $i$ thì Anne sẽ đọc quân bài thứ $n-i+1$. Ta thấy mệnh đề hiển nhiên đúng với $i=0$ vì Anne chưa đọc lá bài nào. Giả sử mệnh đề đúng với $i=k$, xét lần đọc thứ $k+1$. Trong lần này Anne sẽ đọc quân bài thứ $n-k$ trên bảng và bộ bài của Alice đang ở trạng thái $k$. Nếu quân bài Anne đang đọc không nằm cạnh ô trống trong trạng thái $n-k$ thì trong trạng thái $n-k-1$ nó cũng vậy, vì vậy nên Alice sẽ không làm gì và bộ bài sẽ trở về trạng thái $n-k-1$. Nếu quân bài đó nằm cạnh ô trống thì trong trạng thái $n-k-1$ nó cũng nằm cạnh ô trống nhưng ngược hướng so với trạng thái $n-k$, lúc đó Alice sẽ dịch chuyển quân bài đó về ô trống và bộ bài trở về trạng thái $n-k-1$. Chứng minh quy nạp hoàn tất. Vì vậy sau $n$ lần đọc thì bộ bài trở về trạng thái 0. Nhưng trong trạng thái này quân Át nằm cạnh ô trống nên Anne không thể đạt được mục đích. Vậy cho dù Anne có đọc thế nào thì cũng có 1 cách sắp xếp bộ bài sao cho cuối cùng quân Át cũng ở cạnh ô trống

Lời giải bài 11:

Với mỗi số $2\leq a\leq 21$ thì ta sẽ gọi $b$ là 1 số đối của $a$ nếu

-)$b<a$

-)Xét hộp $A$ chứa $a$ viên bi và hộp $B$ chứa $b$ viên bi. Sau một số bước chuyển bi giữa 2 hộp này thì hộp $A$ chứa $b$ viên bi và hộp $B$ có $a$ viên bi.

Nếu $a$ là số chẵn thì $a$ có 1 số đối là $\frac{a}{2}$

Nếu $a$ là số lẻ thì ta sẽ xây dừng cặp $(a;b)$ bới $b$ là số đối của $a$ như sau:

$(3;2),(5;4),(7;4),(9;8),(11;4);(13;4);(15;4);(17;16);(19;8);(21;4)$

Ta sẽ chứng minh bài toán tổng quát hơn:  Với $(a_{1};a_{2};...;a_{n})$ là 1 hoán vị của $(1;2;...;n)$ với $n\leq 21$ thì ta có thể thực hiện chuyển đổi giữa các hộp sao cho hộp thứ $i$ từ trái sang phải chứa $a_i$ viên bi. Mệnh đề đúng với $n=1,2$, giả sử mệnh đề đúng với $n=k<21$, xét $n=k+1$. Giả sử lúc đầu hộp $x$ chứa $k+1$ viên bi và 1 hộp thứ $y$ với $y\leq 21$ bất kì. Khi đó với $k$ hộp còn lại, theo mệnh đề quy nạp thì ta có thể chuyển bi sao cho với $b$ là số đối của $k+1$ thì hộp $y$ chứa $b$ viên bi. Ta thực hiện chuyển đổi liên tiếp giữa 2 hộp này thì cuối cùng hộp $y$ chứa $k+1$ viên bi, hộp $x$ chứa $b$ viên bi. Cho $y$ chạy từ $1$ đến $k+1$ thì lúc đó ta có thể di chuyển bi sao cho tất cả mọi hộp đều có thể chứa $k+1$ viên bi, hoán vị $k$ hộp còn lại ta sẽ tạo ra được tất cả các hoán vị của $(1;2;...;k+1)$. Vậy mệnh đề đúng với $n=k+1$, vì vậy mệnh đề đúng với $n=21$. Vì $(1;2;...;21)$ là 1 hoán vị của $(1;2;...;21)$ nên ta có thể chuyển bi sao cho các hộp chứa số bi thoả mãn

Bài 12: (USAMO 2016)

Cho $n,k$ là hai số nguyên, $n\geq k\geq 2$. Bạn tham gia vào một trò chơi với một phù thuỷ 

Phù thuỷ có $2n$ lá bài, với mỗi $i=1,2,...,n$ thì có đúng hai lá bài cùng được đánh số là $i$. Lúc đầu, phù thuỷ sắp úp tất cả lá bài trên một hàng, thứ tự bất kỳ.
Về phần bạn, bạn sẽ lặp lại các thao tác sau: chỉ vào $k$ lá bài bạn muốn chọn, phù thuỷ sẽ lật ngửa lên hộ bạn. Nếu lật lên, có hai tấm thẻ cùng được đánh 1 số, bạn thắng, game sẽ kết thúc. Nếu ngược lại, phù thuỷ sẽ hoán vị $k$ lá bài đó và lật úp chúng lại. Mỗi lần bạn làm vậy là một bước đi. Và bạn sẽ lặp lại thao tác trên.
Ta gọi game này là thành công nếu tồn tại $m$ nguyên dương nào đó và một chiến thuật nào đó ở tối đa $m$ bước đi, không quan trọng việc phù thuỷ xáo trộn bài của bạn.

Hỏi với $n$ và $k$ nào thì bạn có thể chiến thắng?




#637415 Chứng minh rằng luôn chỉ ra được 3 điểm trên mặt phẳng làm thành tam giác vuô...

Đã gửi bởi JUV on 01-06-2016 - 16:18 trong Tổ hợp và rời rạc

Xét 2 điểm $A,B$ được tô cùng màu trên mp, dựng hình vuông $ABCD$ và giao điểm 2 đường chéo là $G$. Giả sử cả $A,B$ được tô đỏ. Nếu 1 trong 3 điểm $C,D,G$ được tô đỏ thì điểm đó tạo với 2 điểm $A,B$ tam giác vuông cân 3 đỉnh tô cùng màu. Nếu không thì $CDG$ là tam giác vuông ân thoả mãn đề bài




#637762 Marathon Tổ hợp và rời rạc VMF

Đã gửi bởi JUV on 03-06-2016 - 11:08 trong Tổ hợp và rời rạc

Bạn xem lại đề 13 nhé. Trong TH $n=3$ thì $S_{3}=0$;$S_{2}=8$;$S_{0}=12$




#637784 Marathon Tổ hợp và rời rạc VMF

Đã gửi bởi JUV on 03-06-2016 - 13:24 trong Tổ hợp và rời rạc

Đề bài hoàn toàn đúng bạn ạ.

Với $n=3$ thì $S_0=8$ bạn nhé, chỉ có 8 cặp điểm nằm trên đường chéo của các hình chữ nhật $1\times 2$ mới thuộc $S_0$ thôi.

Thế còn 2 cặp điểm $(2;3),(2;1)$ và $(1;2),(3;2)$ có phải là 2 đỉnh của hình vuông nào đâu

P/s: Thật ra mình đếm thì $S_{0}=10$ chứ không phải $12$




#638174 Cmr tồn tại vô hạn các số nguyên x,y,z

Đã gửi bởi JUV on 05-06-2016 - 07:46 trong Số học

Đầu tiên thấy bộ số $(x;y;z)=(1;-1;1)$ thoả mãn, tiếp theo xét 1 bộ số $(x;y;z)$ thoả mãn, có $x^{5}+8y^{3}+7z^{2}=0$. Nhân cả 2 vế với $2^{30}$, ta có $(2^{6}x)^{5}+8(2^{10}y)^{3}+7(2^{15}z)^{2}=0$. Vậy nếu $(x;y;z)$ thoả mãn hệ thì $(2^{6}x;2^{10}y;2^{15}z)$ cũng thoả mãn hệ. Vì vậy từ nghiệm $(x;y;z)=(1;-1;1)$ ban đầu ta có thể lập được vô số nghiệm




#638878 Marathon Tổ hợp và rời rạc VMF

Đã gửi bởi JUV on 08-06-2016 - 10:30 trong Tổ hợp và rời rạc

Lời giải bài 14:

Đánh dấu màu xanh tất cả vị trí mà con châu chấu đầu tiên nhảy được tới, màu đỏ các vị trí mà con thứ 2 có thể nhảy được tới. Gọi $d$ là khoảng cách của 2 con châu chấu, đặt $d^2=2^tq$ ($q$ là số lẻ). Xét 1 thời điểm bất kì con châu chấu đầu tiên ở vị trí $A(x_1;y_1)$, con thứ 2 ở vị trí $B(x_2;y_2)$. Giả sử con châu chấu đầu tiên nhảy từ $A$ đến $C(x_3;y_3)$, ta có $(x_1-x_2)^2+(y_1-y_2)^2=(x_3-x_2)^2+(y_3-y_2)^2=d^2=2^tq$.

Đặt $x_1-x_2=a_1; y_1-y_2=b_1; x_3-x_2=a_2; y_3-y_2=b_2$

Nếu $t$ là số chẵn, đặt $t=2n$, lúc đó cả $a_1;a_2;b_1;b_2$ đều chia hết cho $2^n$, vì vậy $(a_1-a_2)^2+(b_1-b_2)^2$ chia hết cho $2^{2n+1}$ hay $(x_1-x_3)^2+(y_1-y_3)^2$ chia hết cho $2^{t+1}$

Nếu $t$ là số lẻ, đặt $t=2n+1$, lúc đó cả $a_1;a_2;b_1;b_2$ đều chia hết cho $2^n$ nhưng không chia hết cho $2^{n+1}$. Đặt $a_1=2^na; a_2=2^nb; b_1=2^nc; b_2=2^nd$, có $a,b,c,d$ đều không chia hết cho 2 nên $(a_1-a_2)^2+(b_1-b_2)^2=2^{2n}((a-b)^2+(c-d)^2)$ chia hết cho $2^{2n+2}$ hay chia hết cho $2^{t+1}$ ( Vì $(a-b)^2+(c-d)^2$ chia hết cho $4$)

Trong cả 2 trường hợp thì $(x_1-x_3)^2+(y_1-y_3)^2$ đều chia hết cho $2^{t+1}$. Từ đó có thể chứng minh rằng bình phương khoảng cách của 2 điểm xanh bất kì trên toạ độ đều chia hết cho $2^{t+1}$. Vì vậy khoảng cách giữa 2 điểm xanh bất kì đều khác $d$. Vì vậy không có điểm nào trên toạ độ tô cả 2 màu xanh và đỏ. Cho nên con châu chấu thứ 2 không thể nào nhảy đến bất kì điểm nào mà con thứ nhất có thể nhảy đến, tương tự với con thứ nhất. Vì vậy 2 con châu chấu không thể đổi chỗ cho nhau

Bài 15: 

Một ảo thuật gia với $1$ bộ bài có $52$ quân bài, có $4$ loại bài là cơ, chuồn, rô, tép, mỗi loại bài có $13$ quân bài. Đầu tiên nhà ảo thuật sẽ đưa bộ bài cho khán giả xào bài, tất cả các quân bài sau khi được xào lại được đặt úp, chồng lên nhau. Trợ thủ của nhà ảo thuật được quyền xem thứ tự các quân bài sau đó viết $1$ trong $2$ số $0$ hoặc $1$ vào mặt sau mỗi quân bài nhưng không được sắp xếp lại bộ bài và cũng không được nói cho nhà ảo thuật biết thứ tự của bộ bài. Mỗi lượt nhà ảo thuật sẽ đoán loại bài của lá bài trên cùng của xấp bài, sau đó loại nó ra khỏi bộ bài. Nhà ảo thuật cứ tiếp tục như thế trong $52$ lượt. Hỏi có chiến lược nào giúp anh ta đoán được đúng loại bài của ít nhất $30$ quân bài không?




#638882 Marathon số học Olympic

Đã gửi bởi JUV on 08-06-2016 - 10:42 trong Số học

Định lý Dirichlet sẽ chỉ được sử dụng không chứng minh khi dự thi IMO. Còn các kỳ thi khác thì đều không được sử dụng khi không chứng minh lại.

Hình như mấy kì thi khác mình đọc thì định lí Dirichlet được thay tên bằng định lí Pigeonhole nội dung tương tự được sử dụng không chứng minh 




#639193 Đề tuyển sinh chuyên Hà Nội 2016-2017

Đã gửi bởi JUV on 09-06-2016 - 18:07 trong Tài liệu - Đề thi

Câu V:

Giả sử tồn tại $2017$ SHT được sắp xếp $1$ cách thoả mãn nếu bỏ $2$ số bất kì cạnh nhau thì $2015$ số còn lại chia được thành $2$ nhóm có tổng bằng nhau. Gọi $2017$ số được sắp xếp thoả mãn là $2017$ số có tính chất $P$

Vì có $2017$ SHT có tính chất $P$ nên nếu nhân mẫu của các SHT đó lên thì được $2017$ STN có tính chất $P$. Gọi $2017$ số đó lần lượt xếp theo chiều kim đồng hồ là $a_{1};a_{2};...;a_{2017}$. Giả sử trong $2017$ số đó có 1 số chẵn, 1 số lẻ thì vì $2017$ là số lẻ nên lúc đó trên vòng tròn tồn tại $2$ số liền kề cùng tính chẵn lẻ và $2$ số liền kề không cùng tính chẵn lẻ. Vì vậy có thể bỏ 1 trong 2 cặp số đó để tổng $2015$ số còn lại lẻ, lúc đó thì không thể có cách chia $2015$ số còn lại thoả mãn đề bài. Giả sử tất cả các số trên vòng tròn cùng tính chẵn lẻ, $2017$ số đó không thể cùng lẻ vì cho dù bỏ đi $2$ số nào thì tổng các số còn lại đều lẻ nên không thể chia được. Vậy tất cả các số trên vòng tròn đều chẵn. Đặt $a_{i}=2b_{i}$ với $i$ chạy từ $1$ đến $2017$. Vì $2017$ số $a_{1};a_{2};...a_{2017}$ có tính chất $P$ nên $b_{1};b_{2};...b_{2017}$ cũng có tính chất $P$. Lập luận tương tự $b_{1};b_{2};...b_{2017}$ đều chẵn. Tiếp tục đặt $b_{i}=2c_{i}$ và lặp lại vô hạn bước như vậy, ta có $a_{1}=a_{2}=...=a_{2017}=0$ (vô lí vì các số hữu tỉ ban đầu dương)

Suy ra dpcm




#639390 Marathon Tổ hợp và rời rạc VMF

Đã gửi bởi JUV on 10-06-2016 - 17:42 trong Tổ hợp và rời rạc

Mình có một số thắc mắc sau:

- Trợ lý và nhà ảo thuật có được thống nhất chiến thuật từ trước khi xáo bài hay không?

- Có bắt buộc trợ lý phải viết số vào sau lá bài không?

- Điều hiển nhiên là trợ lí phải giúp nhà ảo thuật đạt được mục đích nên cũng phải thống nhất chiến thuật

- Trợ lí bắt buộc phải viết số sau lá bài




#639412 Một Số Bổ Đề, Định lý Số Học

Đã gửi bởi JUV on 10-06-2016 - 18:39 trong Tài liệu, chuyên đề, phương pháp về Số học

Bài 1:

Có $q \equiv 1\pmod{p}$

dễ thấy $p-1$ không chia hết cho $q$ nên $p^{2}+p+1$ chia hết cho $q$.

Có $q \equiv 1\pmod{p}$, $p^{2}+p+1 \equiv 1\pmod{p} $ nên $\frac{p^2+p+1}{q} \equiv 1\pmod{p}$

Vì vậy $\frac{p^2+p+1-q}{q} \vdots p$, vì vậy $p^2+p+1-q \vdots pq$. Dễ chứng minh $p^2+p+1-q<pq$ nên $p^2+p+1-q=0$ vì vậy $p+q=(p+1)^2$ là SCP




#639582 Marathon Tổ hợp và rời rạc VMF

Đã gửi bởi JUV on 11-06-2016 - 14:17 trong Tổ hợp và rời rạc

Cho mình hỏi liệu rằng cần tất cả các lá bài phải đánh dấu không hay lá có lá không?

Tất cả lá bài phải đánh dấu




#642029 Tìm tất cả các số nguyên dương n và k thỏa mãn yêu cầu bài toán

Đã gửi bởi JUV on 24-06-2016 - 18:56 trong Tổ hợp và rời rạc

Xét 1 người $A$ bất kì, người đó sẽ giải 3 bài toán bất kì, mỗi bài toán sẽ có $k$ người giải nó tính cả $A$. Vì mỗi người trong số $n$ người với $A$ có đúng 1 bài toán mà 2 người đều giải được nên trong 3 bài toán mà $A$ giải được thì không có người nào mà giải được ít nhất 2 trong số 3 bài. Nhưng mỗi người trong số $n-1$ người còn lại giải được ít nhất $1$ trong số $3$ bài toán đó nên tồn tại 1 song ánh giữa tập $n-1$ người đã trừ $A$ với tập các người giải được ít nhất $1$ bài toán trong số $3$ bài đã nêu trừ $A$. Vì vậy $n=3(k-1)+1$. Vì mỗi người giải được $3$ bài toán và mỗi bài toán được giải bởi $k$ người nên số bài toán được giải là $\frac{3n}{k}$, vì vậy $k$ là ước của $3n$ hay là ước của $9(k-1)+3$. Vì vậy $k$ là ước của $6$. Tuy nhiên nếu xét $k=6$ thì giả sử người $A$ làm được $3$ bài $1,2,3$ thì trong số $15$ người còn lại có $5$ người làm được bài $1$, $5$ người làm được bài $2$,$5$ người làm được bài $3$. Xét những người làm được bài $4$ thì theo nguyên lí Pigeonhole thì tồn tại $2$ người cùng làm được bài $4$, $2$ người cùng làm được bài $1$ hoặc $2$ hoặc $3$ (vô lí vì $2$ người bất kì thì chỉ làm được 1 bài chung). Vì vậy $(k;n)=(1;1);(2;4);(3:7)$




#642308 18th ELMO (ELMO Lives Mostly Outside)

Đã gửi bởi JUV on 26-06-2016 - 18:57 trong Thi HSG Quốc gia và Quốc tế

Thấy thầy Hùng với bạn Bảo chém hình ác quá nên mình cũng giải 1 câu tổ hợp cho đủ bộ.

Bài 5: Gọi $n$ điểm trong tập $S$ là $A_1;A_2;...;A_n$, xét $n\vdots 2$, đặt $t=\frac{n}{2}$ đầu tiên ta xét các đường tròn đường kính $A_{i}A_{i+t}$ với $i$ chạy từ $1$ đến $t$. Tất cả các hình tròn này đều đôi một giao nhau nên màu của $2$ đường tròn bất kì trong $t$ hình tròn này khác nhau nên cần ít nhất $t$ màu để tô. Cách tô như sau: Gọi $t$ màu là $a_1;a_2;...;a_t$, đầu tiên với mọi $i$ chạy từ $1$ đến $t$, cung $A_{i}A_{i+t}$ được tô bởi màu $a_i$. Sau đó với mọi $k<t$, đường tròn đường kính $A_{i}A_{i+k}$ được tô cùng màu với đường tròn đường kính $A_{i}A_{i+t}$ với $i$ chạy từ $1$ đến $n-k$, với mọi $k>t$ thì đường tròn đường kính $A_{i}A_{i+k}$ được tô cùng màu với đường tròn đường kính $A_{i+k-t}A_{i+k}$. Dễ thấy cách tô này thoả mãn đề bài

Xét $n$ là số lẻ, đặt $n=2t+1$, gọi $a$ là số màu ít nhất có thể tô các đường tròn, ta có $2t<n<2t+2$, theo chứng minh trên thì $t\leq a\leq t+1$ (Vì một cách tô với $n$ điểm bằng $a$ màu thì có thể suy ra $1$ cách tô với $2t$ điểm với nhiều nhất $a$ màu, tương tự với cách tô $2t+2$ điểm thì suy ra $1$ cách tô với $n$ điểm bằng nhiều nhất $t+1$ màu ). Nếu $a=t$ thì lúc đó xét các đường tròn đường kính $A_{i}A_{i}$ với $i$ chạy từ $1$ đến $t$, $t$ đường tròn này đôi một cắt nhau nên chúng được tô bằng $t$ màu khác nhau lần lượt là $a_1;a_2;...;a_n$ và đường tròn đường kính $A_{t+1}A_{n}$ được tô màu $a_1$ vì nó cắt $t-1$ đường tròn kia ngoại trừ đường tròn đường kính $A_{1}A_{t+1}$. Ta xét tiếp các đường tròn $A_{i}A_{i+t+1}$ với $i$ chạy từ $1$ đến $t$, $t$ đường tròn này đôi một cắt nhau nên được tô bằng tất cả $t$ màu đã nêu trên. Vì vậy trong $t$ đường tròn đó có $1$ đường tròn tô màu $a_1$, tuy nhiên đường tròn này cắt $1$ trong $2$ đường tròn đường kính $A_{1}A_{t+1}$ và $A_{t}A{n}$ suy ra điều vô lí vì cả $3$ đường tròn tô cùng 1 màu. Vì vậy $a>t$, nên $a=t+1$. Vì có $1$ cách tô với $n+1$ điểm bằng $t+1$ màu nên từ đó có 1 cách tô với $n$ điểm với $t+1$ màu. Vì vậy số màu ít nhất trong mọi trường hợp là $\left [ \frac{n+1}{2} \right ]$

P/s: ELMO là viết tắt của từ gì hay là tên 1 con rối trong Sesame street vậy?